Top Posters
Since Sunday
t
7
m
6
k
6
F
5
j
5
t
5
j
5
G
5
f
5
a
5
d
5
c
5
A free membership is required to access uploaded content. Login or Register.

Ch38 Environmental Emergencies.docx

Uploaded: 6 years ago
Contributor: BritishGent
Category: Nursing
Type: Other
Rating: N/A
Helpful
Unhelpful
Filename:   Ch38 Environmental Emergencies.docx (117.22 kB)
Page Count: 92
Credit Cost: 1
Views: 118
Last Download: N/A
Transcript
Chapter Environmental Emergencies Unit Summary Upon completion of this chapter and related course assignments students will be able to integrate assessment findings with the pathophysiology of environmental emergencies to formulate a field impression develop a comprehensive treatment plan and manage the environmental emergency patient They will understand the pathophysiology of environmental emergencies such as near drowning temperature-related illnesses bites or envenomations dysbarism injuries electrical injuries and radiation exposure Students will be able to identify various species of spiders snakes and insects that have venomous bites or may result in anaphylaxis They will be able to describe four factors that affect how a person deals with exposures to cold or hot environments as well as explain the four different ways a body can lose heat and ways the rate and amount of heat loss or gain can be modified in an emergency situation Students will be able to discuss lightning injuries including incidence risk factors assessment and treatment They will be able to describe how lightning injuries can alter the cardiac electrical conduction system and autonomic nervous system Students will be able to identify signs and symptoms of temperature-related emergencies They will be able to discuss the incidence risk factors and prevention strategies for drowning Students will be able to demonstrate how to treat and care for patients with environmental emergencies National EMS Education Standard Competencies Trauma Integrates assessment findings with principles of epidemiology and pathophysiology to formulate a field impression to implement a comprehensive treatment disposition plan for an acutely injured patient Environmental Emergencies Recognition and management of Submersion incidents pp - Temperature-related illness pp - Pathophysiology assessment and management of Near drowning pp - Temperature-related illness pp - Bites and envenomations pp - Dysbarism - High-altitude pp - - Diving injuries pp - Electrical injury pp - Radiation exposure see chapter Burns High-altitude illness pp - Knowledge Objectives Describe four factors that affect how a person deals with exposure to a cold or hot environment and how each one relates to emergency medical care p Explain the four different ways a body can lose heat and ways the rate and amount of heat loss or gain can be modified in an emergency situation pp - Describe the various forms of illnesses caused by heat exposure including their signs and symptoms and give examples of persons who are at the greatest risk of developing one of them pp - Describe the process of providing emergency care to a patient who has sustained a heat injury including assessment of the patient review of signs and symptoms and management of care pp - Define and discuss hypothermia including the signs and symptoms of its four different stages and the risk factors for developing it pp - Explain local cold injuries and their underlying causes pp - Describe the process of providing emergency care to a patient who has sustained a local cold injury including assessment of the patient review of signs and symptoms and management of care pp - Explain the importance of following regional and state protocols when rewarming a patient who is experiencing moderate or severe hypothermia pp - Define drowning and discuss its incidence risk factors and prevention pp - Describe the various types of diving emergencies how they may occur and their signs and symptoms pp - Describe the process of providing emergency care to a patient who has been involved in a drowning or diving emergency including assessment of the patient review of signs and symptoms and management of care pp - - Discuss the types of dysbarism injuries that may be caused by high altitudes including their signs and symptoms and emergency medical treatment in the field pp - Discuss lightning injuries including their incidence risk factors assessment and emergency medical treatment pp - Identify the species of spiders found in the United States that may cause life-threatening injuries and then describe the process of providing emergency care to patients who have been bitten by each type pp - Discuss the emergency medical care of patients who have been stung by hymenoptera and scorpions and bitten by ticks including steps the paramedic should follow if a patient develops a severe reaction to the sting or bite pp - - Identify the species of snakes found in the United States that are venomous and then describe the process of providing emergency care to patients who have been bitten by each type and are showing signs of envenomation pp - Discuss the emergency medical care of a patient who has been bitten by a tick and of a patient who experiences paralysis thereafter pp - Skills Objectives Demonstrate the emergency medical treatment of local cold injuries in the field pp - Demonstrate using a warm-water bath to rewarm the limb of a patient who has sustained a local cold injury pp - Demonstrate how to treat a patient with heat cramps pp - Demonstrate how to treat a patient with heat exhaustion pp - Demonstrate how to treat a patient with heatstroke pp - Demonstrate how to care for a patient who is suspected of having an air embolism or decompression sickness following a diving emergency pp - Demonstrate how to care for a patient who has been struck by lightning pp - Demonstrate how to care for a patient who has been bitten by a black widow or brown recluse spider pp - Demonstrate how to care for a patient who has been bitten by a pit viper and is showing signs of envenomation pp - Demonstrate how to care for a patient who has been bitten by a coral snake and is showing signs of envenomation pp - Demonstrate how to care for a patient who is experiencing altitude illness pp - Demonstrate how to care for a patient who has been bitten by a tick pp - Readings and Preparation Review all instructional materials including Chapter of Nancy Caroline s Emergency Care in the Streets Seventh Edition and all related presentation support materials Consider reading the following articles Environmental Emergencies by W Gandy K Grayson EMS World June online at http www emsworld com article environmental-emergencies Unusual Resuscitation Situations Environmental Emergencies by G Friese R Wojciehoski EMS World September online at http www emsworld com article unusual-resucitation-situations-environmental-emergencies Surviving Drowning by C Krin EMS World April online at http www emsworld com article surviving-drowning Treating Heat-Related Illness in the Elderly by J D Angelo EMS World August online at http www emsworld com article treating-heat-related-illness-in-the-elderly Diving Emergencies by D Wohl EMS World June online at http www emsworld com article diving-emergencies Along Came a Spider by R Brouhard EMS World March online at http www emsworld com article along-came-a-spider EMS Recap High-Altitude Illnesses by R Sippel EMS World March online at http www emsworld com article ems-recap-high-altitude-illnesses Stings and Bites What to Do about Envenomation Injuries by D Powers EMS World April online at http www emsworld com article stings-and-bites-what-to-do-about-envenomation-injuries A Shocking Call by P Murphy C Colwell G Pineda T Bryan EMS World February online at http www emsworld com article a-shocking-call If your area has environmental challenges that result in specific environmental emergencies obtain specific local EMS agency protocols or equipment for dealing with these emergencies Develop patient scenario cards for use in group exercises later in the course Incorporate a variety of environmental emergencies including specific bites and envenomations signs symptoms age groups and environmental factors Support Materials Lecture PowerPoint presentation Case Study PowerPoint presentation Identify pictures of local reptiles spiders and other insects or animals that may cause bite or envenomation injuries If your area has specific equipment used to address environmental injuries and access to patients request that the local EMS agency participate in scenario practice with your class Enhancements Direct students to visit the companion website to Nancy Caroline s Emergency Care in the Streets Seventh Edition at http www paramedic emszone com for online activities Contact a local hyperbaric clinic to identify a guest speaker that can address the application of hyperbaric therapy or arrange a tour of the facility for students to view the devices Contact your local animal control division to identify a guest speaker that can speak to the class about reptiles that are indigenous to your area Contact a local pest control organization to identify a guest speaker that can identify hymenoptera scorpions and spiders that are indigenous to your area Content connections Review physiological considerations for treatment of patients with hypothermia-related illnesses and use of electrical or pharmacological therapy as well as necessitity of rewarming patients before determination of death Review trauma care in consideration of the patient with a hypothermic emergency and the significance of warming the patient in supporting shock care Review the importance of scene safety when entering scenes that might be habitats for hymenoptera scorpions spiders and snakes Remind students not to set equipment in these areas and to check bags and devices thoroughly when clear of the scene Cultural considerations Remind students of the increased risk to pediatric and geriatric patient populations for hypothermia in otherwise moderate temperatures Review with students that some religious groups use spiders and the importance of treating the patient without criticizing religious practices Teaching Tips This is an important section to remind students of the importance of personal crew and bystander safety when dealing with environmental emergencies Encourage students to use caution at scenes where venomous snakes may be present Discourage students from attempting water rescues if not specifically trained for this type of activity Unit Activities Writing activities Assign students an environmental emergency Have them research the topic to include a comparison of incidence seen local to their community statewide and nationally Students should submit an essay that summarizes their findings and prevention efforts to address the type of emergency assigned Student presentations Assign students an environmental emergency including specific bites and envenomation emergencies Have them develop a presentation of the emergency and prepare a prevention guide that might be shared within their community Have them include information for first aid care in the community prior to EMS arrival Group activities Using scenario cards developed prior to the lesson divide students into groups of four or five to discuss scenarios and treatment options Students should be able to identify environmental factors that can prevent access to the patient delay definitive treatment or present challenges to transport of the patient as applicable to the scenario Visual thinking Using pictures collected prior to this lesson have students identify common reptiles scorpions hymenoptera and spiders as well as treatment for bites or envenomations Include images of common habitats where these creatures might be found for students to identify Pre-Lecture You are the Medic You are the Medic is a progressive case study that encourages critical-thinking skills Instructor Directions Direct students to read the You are the Medic scenario found throughout Chapter You may wish to assign students to a partner or a group Direct them to review the discussion questions at the end of the scenario and prepare a response to each question Facilitate a class dialogue centered on the discussion questions and the Patient Care Report You may also use this as an individual activity and ask students to turn in their comments on a separate piece of paper Lecture I Introduction A Environmental emergencies Medical conditions caused or worsened by weather terrain or unique atmospheric conditions present at high altitude or underwater From to people in the United States died of hypothermia-related causes Exposure to extreme heat caused deaths between and In the summer of a heat wave in Europe caused as many as deaths B EMS providers are challenged to recognize patients with environmental emergencies in Endurance sports events Mass gatherings Acutely confused older patients C Environmental conditions can directly cause harm or complicate treatment Weather conditions can affect body s ability to adapt to the environment a Wind Examples a Hikers experiencing cold illnesses during summer rainstorms b Overdressed snow sport participants experiencing heat illnesses during strenuous outings Locations of emergencies may have impact on a Learning about problem b Responding to the scene c Reaching patients in remote settings D Common risk factors predispose people to environmental emergencies Young and older people harder to maintain thermoregulation Some conditions alter the body s ability to compensate for environmental extremes such as a Diabetes b Cardiac disease c Restrictive lung disease d Thyroid disease e Psychiatric illnesses Dehydration causes risk in environmental emergencies Overall health and fitness is a factor II Anatomy and Physiology A Homeostasis and body temperature Homeostasis Body processes that balance supply and demand of the body s needs Thermosensitive neurons in the anterior hypothalamus ensure balance between heat production and dissipation thermoregulation a Hypothalamus works on the negative feedback control principle i Rise in core body temperature causes responses that increase heat loss and turn off heat production pathways ii Fall in core body temperature causes heat production and conservation turning off heat-liberating pathways The human body tries to keep a constant temperature of F C a Balance between heat produced or absorbed by the body and heat released to outside b Core temperatures remain relatively constant c Periphery temperatures in the skin and extremities fluctuate giving it a major role in thermoregulation d Lowest known survivable body temperature in accidental hypothermia is F C e Core body temperatures CBT defined as temperature in the part of the body comprising the heart lungs brain and abdominal viscera f Starting temperatures for i Hypothermia F C ii Heat stroke F C Oral temperature commonly used for measurement of general medical conditions a Can vary dramatically from CBT if patient had been i Breathing by mouth ii Drinking hot or cold liquids b Axillary temperature in the armpit about F cooler c Rectal temperature about F hotter i Rectal thermometers usually most accurate means of determining CBTs but CBTs are highly unlikely to impact treatment d Tympanic temperatures from devices measuring heat reflected off eardrum and skin temperatures are less reliable B Thermoregulatory mechanisms Main thermoregulatory center is located in specialized tissue in the hypothalamus a Thermogenic tissues mediated by sympathetic nervous system b Thermolytic tissues mediated by parasympathetic nervous system c Hypothalamus receives signals from i Peripheral warm and cold thermoreceptors in the skin and muscles ii Central receptors in the core triggered by changes in blood temperature At rest body produces heat by metabolism of nutrients with liberation of water and carbon dioxide a Basal metabolic rate BMR heat energy produced at rest from normal body metabolic reactions i Liver and skeletal muscles major contributors b BMR of average -kg person is around to kcal per hour based on factors including i Age ii Gender iii Stress iv Hormones c As ratio of body surface area to body volume increases heat loss to environment increases i With two people of the same weight the shorter will have a higher BMR Exertion affects the metabolic rate a A brisk walk can produce heat of kcal h b The recommended daily caloric intake is to kcal c Men need slightly higher caloric intake than women i A moderately active woman between ages and to calories ii A man in the same category to calories Some heat generated by metabolism and glycogen breakdown is used to warm the body a The excess is dissipated by taking advantage of the temperature gradient between the body and the outside environment b If the environmental temperature is higher the body is also heated by absorption of the outside heat i Bright sunshine on a hot breezeless day can add up to kcal h to the internal heat load Skin has a vital role in body temperature regulation a Body can both conserve and liberate heat energy through the skin i To liberate heat blood flow to the skin can include up to liters per minute and of cardiac output ii In cold blood flow can approach zero in certain areas C Physiologic responses to heat and cold Thermolysis a Body reacts to daily heat energy production and hot environment in the same way b Thermolysis release of stored heat and energy from the body c Increase in the CBT causes the hypothalamus to send signals through the autonomic nervous system causing vasodilation and sweating d Because of cutaneous vasodilation i The effective volume of the vascular system is increased ii The heart increases its output to compensate for this effect iii The pulse rate and stroke volume increase iv The work of the heart is markedly increased e If vasodilation increases dramatically i There may be a complete loss of vasomotor control the ability of the arteries to constrict in response to sympathetic stimulation a Blood pools in the periphery b The patient could go into neurogenic shock f When warmed blood from the core and overheated muscles heads for peripherally dilated cutaneous vessels it may be cooled by i Radiation transfer of heat via electromagnetic waves a Accounts for more than of heat loss in a cooler setting b Especially heat loss through the head c If ambient temperature is high F or greater body heat will be gained ii Conduction Transfer of heat from a hotter to a cooler object by direct physical contact a Air is a poor conductor of heat only of body heat lost b Ground is a good conductor c Water is the best conductor A person in a cold lake will lose heat times faster than a dry person exposed to air of the same temperature Soaked clothing can be just as dangerous iii Convection transfer of energy that takes place when moving air or liquids disturb molecules next to an object a Aids in conduction b Example Blowing on hot food c Example Air moving across body surface picks up heat and carries it away The faster the air moves the faster the heat is removed from the body Wind chill factor measures the effect of a given temperature at a given wind speed iv Evaporation the conversion of a liquid to a gas liberates kcal per mL of sweat a Sweating heat dissipation by evaporation accounts for of cooling b Main mode of cooling in higher temperatures until high humidity levels slows the rate of evaporation c Minor role via respiration d Behind the evaporative method of cooling for heatstroke patients e In cold conditions wet clothes cause heat loss by conduction and further heat loss by evaporation as the clothes dry g These mechanisms require a thermal gradient between the body and its surroundings i When the outside temperature approaches or exceeds skin surface temperature a Heat loss by radiation and convection diminishes and then ceases b The body absorbs heat c Increase in blood flow to the skin promotes increased heat absorption h The body can only dissipate heat by sweat evaporation when ambient temperature approaches body temperature and then only to a point i A healthy adult can sweat a maximum of about L h but cannot maintain that rate for more than a few hours ii The ambient air must be relatively unsaturated with water for effective sweat evaporation iii As relative humidity increases rate of evaporation decreases effective sweat evaporation ceases at a relative humidity of Thermogenesis a The production of heat and energy for the body and the main method of dealing with cold stressors b To help create body heat the hypothalamic center and sympathetic nervous system can i Increase muscle tone in the short term ii Increase thyroid levels in the long term c Hypothalamus also stimulates peripheral vasoconstriction to shunt blood to the core d The eccrine sweat glands receive cholinergic stimuli to decrease sweating e Heavier people are more insulated from cold f Conservation of heat for the core continues until the ability to generate heat is overwhelmed causing hypothermia III Pathophysiology Assessment and Management of Heat Illness A Heat illness An increase in CBT because of inadequate thermolysis Inability to rid heat buildup from the body often because of a Hot and humid conditions b A person s i General state of health ii Clothing iii Mobility iv Age v Preexisting illnesses vi Certain medications When thermoregulatory system fails the CBT soars a Can rise from normal to F C in less than minutes as in heatstroke B Risk factors for heat illness Older people are at particular risk because they do not adjust as well to heat a Perspire less b Acclimatize more slowly c Feel thirst less readily when dehydrated d Have decreased mobility which may affect ability to get fluids e Are more likely to have chronic conditions that interfere with normal heat regulation f Are more likely to be taking medications that disrupt the body s ability to dissipate heat Medications can affect the body s ability to regulate temperature a Diuretics may cause dehydration and electrolyte disturbances i Conditions may interfere with peripheral vasodilation b Beta blockers can slow tachycardic response to heat stress i Normal age-related decreases in maximum pulse rate can also affect this Acclimatization has numerous adaptive physiologic effects a Can decrease likelihood of heat illness b Takes days of controlled progressive exertion in a hot environment to be effective In young and healthy people infants and young children are most vulnerable to heat stress a Have proportionately higher metabolic heat production b Have a CBT that rises faster during dehydration c Do not dissipate heat as well because of smaller organ and vascular systems Also at risk are athletes and military recruits engaged in heavy exertions in hot conditions C Heat cramps Pathophysiology a Acute involuntary painful muscle spasms usually in the lower extremities the abdomen or both b Occur because of i Profuse sweating ii Sodium loss from sweating c Factors contributing to heat cramps i Salt depletion ii Dehydration iii Muscle fatigue d Most often afflict those in good physical conditions i Athletes ii Military personnel iii Physical laborers e A recent study of US college football players twofold increase in sweat sodium losses in athletes prone to heat cramps f A person sweating heavily while exerting himself or herself in a hot environment is losing both fluids and salt i Plain water will not replace the lost salt ii Rehabilitation sector at a fire should keep approved electrolyte solutions available Assessment a Usually start during strenuous and or prolonged physical activity i May be mild with a Slight abdominal cramping b Tingling in extremities ii Usually present with a Severe incapacitating pain in extremities and abdomen b Possible hypotension and nausea c Alertness d Rapid pulse e Pale and moist skin f Normal temperatures Management a Treatment to eliminate exposure and restore lost salt and water i Move patient to a cool environment ii Have him her lie down if feeling faint iii If no nausea give one or two glasses of a salt-containing solution and have him her drink it slowly a Give him her salty chips or pretzels b Salt tablets may irritate the stomach lining or worsen nausea iv If too nauseated to take liquids by mouth a Rapidly infuse normal saline by IV b Consult medical control for IV rate v Do not massage cramping muscles may aggravate the pain vi Patient may want to resume activity as the symptoms abate decision should be made by medical control D Heat syncope Pathophysiology a An orthostatic syncopal or near-syncopal episode typically occurring in nonacclimated people under heat stress b Elderly are at higher risk but check that syncope is not cardiac syncope or atypical acute coronary syndromes c Can occur from i Prolonged standing ii Standing suddenly from a sitting or lying positions d Peripheral vasodilation is one of the body s thermoregulatory functions i Syncope possibly caused by gravity pooling blood in the lower body ii Possibly exacerbated by dehydration and or hypokalemia Assessment and management a Treatment involves i Placing patient in supine position ii Replacing fluid deficits b If patient does not recover quickly suspect i Heatstroke ii Heat exhaustion iii Cardiac syncope iv Atypical acute coronary syndromes E Heat exhaustion Pathophysiology a A milder form of heat illness on a continuum leading to heatstroke b Hallmarks are i Volume depletion ii Heat stress c Two classical forms i Water depleted occurs mostly in a Geriatric patients because of Immobility Medications contributing to dehydration Decreased thirst sensitivity b Active younger workers or athletes who do not adequately replace fluids while in a hot environment ii Sodium depleted a May take hours or days to develop b Results from huge sodium losses from sweating but replacing only water d Exercise-associated hyponatremia closely related to sodium-depleted heat exhaustion i Studies show a common thread prolonged exertion in a hot environment along with excessive fluid intake a Too much water in the body in relationship to total sodium ii Evidence points to arginine vasopressin AVP as a contributing factor a A hormone that increases water absorption in the kidneys iii First symptoms often nonspecific and include a Nausea b Vomiting c Weight gain d Headache iv As it becomes more severe mental changes may occur v If untreated could lead to a Cerebral edema b Pulmonary edema c Respiratory distress d Seizures e Coma f Death e Remain alert for i Older debilitated patients who may be hyponatremic without exercise-induced component ii Those participating in extreme endurance sports of more than hours duration f The Second International Exercise-Associated Hyponatremia Consensus Development Conference recommends i Immediate treatment of exercise-associated hyponatremia encephalopathy with a bolus infusion of mL of NaCl to acutely reduce brain edema ii If no clinical improvement two additional boluses should be administered at minute intervals iii Available means of measuring serum sodium on site at athletic events Assessment a Symptoms may be nonspecific and include i Headache ii Fatigue iii Weakness iv Dizziness v Nausea vi Vomiting vii Abdominal cramping b Other signs and symptoms include i Profuse sweating ii Pale and clammy skin iii Core temperature normal or slightly elevated less than F iv Tachycardia response may be blunted if patient is taking beta blocker v Tachypnea that may produce signs of hyperventilation a Carpopedal spasm b Perioral numbness c Low end-tidal carbon dioxide level vi Fast and shallow respirations vii Decreased blood pressure from a Peripheral pooling of blood b Volume depletion viii If blood pressure is not decreased at rest will drop when patient tries to sit or stand from a recumbent position orthostatic hypotension ix If urine is brown suspect rhabdomyolysis c Heat exhaustion may be misdiagnosed as summer flu d If left untreated it may progress to heatstroke e Elderly patients with primary GI infections may be mistaken for having heat exhaustion i Be cautious GI symptoms may also be indicative of heatstroke Management a Treatment is aimed at removing patient from exposure to heat and restoring fluid and electrolyte balance i Move to a cool environment removing excess clothes and placing supine with legs elevated ii If temperature is elevated a Sponge spray or drip with tepid water and fan gently b Heroic measures to lower body temperature rapidly may chill the patient causing shivering and thermogenesis iii Consider cooling chairs for hand and forearm immersion in cold water at fire scenes mass gatherings and endurance sports iv Provide oral hydration with sports drinks a If nausea and vomiting start a normal saline IV line and draw blood to check electrolytes b Use pulse rate and blood pressure to guide the amount of fluids given v If exercise-associated hyponatremia is suspected do not give fluids by mouth a Draw blood to check the blood sodium level b Administer IV normal saline or hypertonic saline vi Monitor cardiac rhythm vital signs temperature and end-tidal carbon dioxide vii If unsure whether heat exhaustion or heatstroke treat for heatstroke a Consider administration of an antiemetic F Heatstroke Pathophysiology a Least common heat illness but most deadly b Caused by severe disturbance in body s thermoregulation c A profound emergency with mortality rates as high as in treated patients and to in untreated patients d Findings to determine heatstroke i Core temperature of more than F C ii Altered mental status e Consequences are related to effects of elevated temperatures on the body s cells as evidenced by i Disruption of cell membranes ii Adenosine triphosphate transport channels iii Enzymes iv Breakdown of muscle cells seen by presence of a Elevated serum creatine phosphokinase b Muscle protein myoglobin in the urine v Electrolyte disturbances vi Breakdown of heat-labile proteins leading to edema and hemorrhage a This increased vascular permeability leads to Decreased cardiac output Hypotension Shock vii Loss of sweating a consequence not a cause f Two heatstroke syndromes i Classic heatstroke passive heatstroke a Usually occurs during heat waves b Most likely to strike very old very young or bedridden people c Most at risk are those With chronic illness diabetes heart disease With alcoholism Taking certain medications diuretics sedatives anticholinergics d High environmental temperatures initially elicit thermolysis but CBT then soars causing symptoms of heatstroke e Greatest risk factors of death include Being confined to a bed Not leaving home daily Not being able to care for oneself f Preexisting psychiatric illness tripled risk of death followed by cardiovascular and pulmonary diseases ii Exertional heatstroke typically an illness of young and fit people exercising in hot and humid conditions a When ambient temperature approach body temperature radiation and convection are no longer effective b If relative humidity rises above evaporation is ineffective c Those that keep exercising in these conditions continue to generate heat with no means to release it d They often sweat profusely causing heat to build within the body as the CBT rises e Common with High school and college athletes Military recruits Participants in intense prolonged activity in hot and humid conditions Assessment a Both types present with similar signs and symptoms b Patients may not be able to give a coherent history because they may be confused delirious or comatose c Earliest signs which are often changes in behavior that may mislead EMS workers into believing they are having a behavioral or substance-related emergency include i Irritability ii Combativeness iii Signs of hallucination d Older patients with heatstroke may have signs resembling stroke including trouble walking talking or using an arm or leg e Patients may also have seizures and constricted pupils f Diagnostic vital sign is a markedly elevated temperature usually greater than F C i May be lower never assume F is required to be a heatstroke g Signs of hyperdynamic state usually present i Tachycardia ii Hyperventilation with lowered end-tidal carbon dioxide iii Lowered peripheral vascular resistance from body trying to cool itself with vasodilation h Characterized by some degree of dehydration i Decreases the body s ability to get hotter core blood to the periphery for thermolysis ii Blood pressure may be normal or decreased depending on dehydration i Skin can be i Classic heatstroke dry red and hot ii Exertional heatstroke pale and sweaty j Presence of sweat is not important in diagnosis k Diagnosis easy to miss i May develop rapidly if heat exhaustion was mistaken for flu ii May present as coma of unknown origin l Assess temperature and keep heatstroke in mind during hot months of the year Fever and conditions that mimic heatstroke a Fevers may be a perplexing challenge i Example Is a nursing home patient s temperature elevated from heatstroke febrile illness or sepsis a Neurologic changes could be present in all three ii History may suggest causes a Example Change in urine color and recent complaint of cough and dyspnea might point to infection instead of heatstroke b Fever can signal a fight against infection by inhibiting reproduction of harmful toxins i Pyrogens act on the hypothalamus by increasing the thermal set point resulting in fever ii The body s thermoregulatory tools maintain the new temperature setting iii Patient may adapt to this change by wearing more clothes or the body may create more heat via shivering iv Aspirin and nonsteroidal anti-inflammatory drugs can lower fever they are dangerous when treating heat illnesses c Anticholinergic poisoning presents with i Elevated temperature ii Dry red skin iii Mental status changes iv Tachycardia v Dilated pupils patients with heatstroke usually have constricted pupils d Rare syndromes that cause hyperthermia i Neuroleptic malignant syndrome NMS caused by antipsychotic and some antiemetic medications causing presentations of a Hyperthermia b Muscular rigidity c Altered mental status d Hyperdynamic state ii Malignant hyperthermia a result of common anesthesia medications notably succinylcholine a Presentation similar to NMS b Researchers are studying a common genetic contributor to malignant hyperthermia and heatstroke Management a If unsure of cause of elevated temperature treat for heatstroke because of its deadly consequences of misdiagnosis i Online medical control may help with treatment plans b Treatment includes i Removing patient from offending environment ii Rapid cooling by a Ice water or cold water immersion b Evaporative cooling by spraying cold or cool water accompanied by blowing fans to promote convection iii In exertional heatstroke ice water immersion more effective in rapidly cooling patients than cold water iv Ineffective methods include a Ice packs alone or in combination with fanning b Fanning without use of water c Cooling blankets v Concerns about shivering and raising CBT have been shown to be unfounded vi Evidence-based guidelines from the American College of Sports Medicine indicate the importance of cooling before transport vii Limitations include a Need for ice b Responsive patients not tolerating the treatment c Summary of treatment steps i Evaluate ABCs a Administer supplemental oxygen b Consider sedation if agitated or combative c Be prepared to intubate ii Move patient to a cool environment and strip the patient a Continue cooling efforts until rectal temperature falls below F C iii Cool as rapidly as possible by quickest means possible a Ice water immersion is fastest cooling method Ice water-soaked blankets and fanning are nearly as effective Monitor airway status and seizure activity b Spray with cold or cool water while fanning constantly Ambulance should carry a portable fan during the summer months or Use a fire department ventilation fan iv Start an IV line a Administer normal saline b Check blood glucose level c Be careful with fluids pulmonary edema is a complication of heatstroke d Cooling promotes peripheral vasoconstriction which may raise blood pressure v Monitor cardiac rhythm a Rhabdomyolysis can occur resulting in hyperkalemia vi Prepare to treat for seizures with common antiseizure medications a Lorazepam b Midazolam c Diazepam Prevention of heat illness a Military organizations were the first to understand importance of acclimatization b Modern contributions include standards for conducting training in hot environments c Heat stress indices are a variety of scientific measurements that allow for estimation of effect of variables on the ability to work in hot environments these variables include i Temperature ii Humidity iii Wind speed d Indices regulated for military numerous industries and scholastic sports i Wet-bulb globe temperature index is most widely used and takes into account a Wet bulb temperature measured by placing moistened material over the end of the thermometer while exposed to sun or wind b Air or dry bulb temperature shielded from the sun c Black globe temperature thermometer inside a small black ball to estimate the effects of direct sunlight e Measures to protect from heat illness include i Acclimatize whenever possible and maintain personal fitness ii Limit time spent in heavy activity in PPE especially during hottest parts of day and season a Paramedics should have appropriate summer uniforms iii Maintain hydration eating appropriately and resting a Avoid high-sugar and or caffeinated beverages b If ambulance does not have onboard refrigerator carry a portable cooler stocked with cold sports drinks or other salt-containing drinks iv Develop or research professional organizations standards on activities in hot weather v Improve physical fitness in cardiovascular endurance and muscular strength vi Conduct community-based programs aimed at high-risk populations f Be alert for early symptoms of heat illness such as i Headache ii Nausea iii Cramps iv Dizziness IV Pathophysiology Assessment and Management of Cold Injuries A Local cold injury frostbite Pathophysiology a Most cold injuries are localized to extremities or exposed body parts i Ears ii Nose iii Upper cheek iv Tips of fingers or toes b Local freezing injuries usually called frostbite i An ischemic injury classified as superficial or deep c Frostnip is a mild form of frostbite i Develops slowly and is usually not painful although the patient may report numbness ii Common among those participating in winter sports iii Easily treated by placing a warm hand firmly over the affected area a If fingers are affected warm them by placing them in the armpit iv Return of warmth is signaled by redness and tingling v Windmilling Warming technique that involves rapidly making a large circle with your arm and hand to force blood into the cold hand a Start with hand next to the side b Raise it backward and up until reaching straight up c Move it rapidly down frontward d Deeper degrees of frostbite involve freezing of tissues i Can occur in ambient temperatures well below freezing point ii Cascades starts with freezing of tissues with microvasoconstriction and local fluid shifts iii When temperatures are low enough the water in cells turn to ice crystals causing a Cellular shrinkage b Hyperosmolar state iv Further complicated by increased viscosity accompanied by a Sludging b Poor flow c Capillary leakage from osmotic imbalance d Resultant thrombus and ischemic injury e Risk factors for frostbite include i Cold exposure without adequate clothing ii Impeding circulation to extremities a Wearing restrictive or tight clothing b Smoking constricts arteries c Drinking alcohol causes diuresis and helps peripherally dilate blood vessels iii Fatigue dehydration or hunger iv Coming in direct contact with cold objects conduction v Hypothermia most likely way to sustain a local cold injury Assessment a Superficial frostbite An altered sensation of numbness tingling and burning i Skin is a White and waxy b Firm to palpation but underlying tissues are soft ii Once thawing occurs a Injured area turns cyanotic b Experiences hot stinging sensation iii Capillary leakage produces edema blisters more characteristic of deep frostbite iv Dull or throbbing pain may persist for days or weeks b Deep frostbite usually involves hands or feet i Frozen extremity may look a White b Yellow-white c Mottled blue-white ii It will be a Hard b Cold c Without sensation iii Major damage occurs when tissues thaw out especially if thawing is gradual a Partial refreezing of melted water may occur b New ice crystals tend to be much larger and cause greater tissue damage iv As thawing occurs a Injured area turns purple and excruciatingly painful b May cause gangrene within a few days Management a Difficult to determine depth of injury when first seen b As area is warmed reperfusion injury can cause excruciating pain c Distance to hospital and whether injured extremity has been partially or completely thawed before paramedic arrival determines treatment i If extremity is still frozen you may prefer to leave it frozen until arrival at the hospital ii Rapid rewarming is difficult in the field iii Contact medical control for options d General principles i Get patient out of the cold and remove wet clothing ii Do not rub or massage the frostbitten area as it will cause further damage iii Transport with injured area elevated iv Administer pain medication as needed v Cover blisters with dry sterile dressing vi Consider rewarming only if the potential to refreeze does not exist e Principles of rewarming i If medical control agrees rewarm the injured extremity before transport a Use a water bath make sure the extremity can be immersed without touching the container b Heat water in a second container then stir into the water bath until temperature is between F and F F and F c While heating water administer IV analgesia fentanyl or morphine because the rewarming will cause severe pain ii When the water is at the appropriate temperature gently immerse the injured extremity a If water temperature falls below F C temporarily remove the extremity while adding more hot water b Stir the water and keep adding hot water until the bath reaches the appropriate temperature range c Re-immerse the extremity iii Rewarming procedure usually takes to minutes a Complete when the frozen area is warm to the touch and is deep red or bluish and remains red when removed from the water bath b Keep the patient warm indoors if possible with insulated clothing and blankets c Do not allow the patient to smoke because nicotine interferes with blood flow to the injured area d Do not allow the patient to ingest anything by mouth iv Once rewarmed dry the extremity and gently apply sterile dressings a Use sterile gauze to separate frostbitten fingers and toes B Trench foot and chilblains Trench foot A process similar to frostbite but can occur at temperatures as high as F a Caused by prolonged exposure to cool wet conditions b Can be explained by conduction i Wet feet lose heat times faster than dry feet ii Vasoconstriction and an ischemic cascade similar to that of frostbite set in c Prevention is the best treatment i Keep feet dry and warm Chilblains Itchy reddish or purple lesions on the face or extremities a Believed to represent longer exposure to temperatures just above freezing b Treatment includes i Removal from environmental extremes ii Room temperature rewarming C Hypothermia Pathophysiology a Defined as a decrease in CBT generally starting at F C owing to inadequate thermogenesis and or excess environmental cold stress i Pathophysiologic mechanisms start sooner than F C ii Some experts define hypothermia as below F C b Extreme cold weather does not need to be present i Example A geriatric stroke patient with alcoholism can become hypothermic in a house heated to F ii Example An unprepared hiker caught in summer wind and rainstorm or person submerged in icy water c Sometimes called accidental hypothermia to distinguish from induced hypothermia used in comatose patients with return of spontaneous circulation d Body regulates cold stress by i Increasing thermogenesis ii Decreasing thermolysis iii Pursuing adaptive behavioral changes e Vasoconstriction produces peripheral tissue ischemia i Continued temperature drops cause the hypothalamic center to stimulate shivering ii If cold continues vasoconstriction is lost and vasodilation occurs causing core heat loss to the periphery Risk factors for hypothermia a Those at risk for hypothermia have i Increased thermolysis ii Decreased thermogenesis iii Impaired thermoregulation iv Other contributing factors b Issues leading to development of hypothermic conditions include i Cold temperatures ii Fatigue iii Improper gear for adverse conditions iv Wetness v Dehydration vi Malnutrition vii Length of exposure viii Intensity of weather conditions c Alcohol is most common cause of heat loss in urban settings by hindering the body s attempt to create insulation around the warm core i Impairs shivering thermogenesis decreased thermogenesis ii Promotes cutaneous vasodilation increased thermolysis iii Can cause liver disease which leads to inadequate glycogen stores iv Causes subnormal nutritional status v Impairs judgment leading to inappropriate behavior in cold conditions d Impaired thermoregulation can also occur by interfering with CNS-mediated vasoconstriction with therapeutic use or overdose of i Sedative medications ii Tricyclic antidepressants iii Phenothiazines e Older patients cannot maintain heat because i Bodies cannot generate heat effectively because of a Reduced muscle mass b Diminished shivering response ii Atrophy of subcutaneous fat cells reduces an elderly patient s insulation against heat loss iii Medications commonly prescribed may interfere with vasoconstriction iv Hypothyroidism and malnutrition may lead to decreased thermogenesis f Trauma may be an important contributing factor to hypothermia i Hypotension and hypovolemia can interfere with normal thermoregulation ii Some injuries may hamper the body s ability to create a shivering response a CNS trauma b Shock c Burns iii Hypothermia in trauma patients can lead to a Lethal coagulation problems b Acidosis g Ensure the ambulance is preheated i Ask if the patient is cold ii Conserve the patient s body heat with blankets Assessment a National Institutes of Health public awareness campaign for hypothermia focusing on the umbles as indicators of how cold affects the cerebral and cognitive functioning in early stages of hypothermia i Stumbles ii Mumbles iii Fumbles iv Grumbles b The American Heart Association ACLS guidelines define i Mild hypothermia CBT of greater than F C ii Moderate hypothermia CBT between F to F C to C iii Severe hypothermia CBT below F C c In early stages the CBT is more than F but patient shows obvious symptoms i Body may compensate through thermogenesis until a Patient can find a way to increase heat production b Glycogen energy stored in liver and muscles is exhausted d Also classified according to time to onset i Acute occurs rapidly e g cold water submersion ii Subacute over a short period of time cold conditions exposure for a short time iii Chronic over days urban homeless person e Another classification i Primary hypothermia caused by cold exposures ii Secondary hypothermia caused by problems such as severe sepsis f In mild hypothermia i Hypothalamic-induced shivering ii Umbles are noticeable iii Initial symptoms are vague a Older people may simply Have a more flat affect Have slightly more confusion Develop symptoms suggestive of stroke including dysarthria and ataxia g No strong correlation has been seen between a specific CBT and signs and symptoms h Different body systems react differently in the slowdown that hypothermia brings about i Most dramatically apparent in the CNS where everything slows a Thinking b Feeling c Speaking slow and possibly slurred d Typically apathetic and shows impaired reasoning ability e Impaired coordination with an ataxic gait f Signs and symptoms may closely resemble Stroke Head injury Acute psychiatric disturbance Alcohol intoxication ii Hypothermia induces changes in the cardiovascular system a Peripheral vasoconstriction shunts blood to the body core b Body s volume receptors interpret increased flow as a volume increase c Sodium reabsorption mechanism becomes impaired leading to cold diuresis d At the same time tissue cooling induces flow of water from intravascular to extravascular spaces e Net effects Increase in blood viscosity Impaired circulation State of hypovolemia iii Meanwhile heart is affected by body temperature drop a Cold speeds up heart initially then slows the rate and disrupts the electric conduction system b At a CBT of F C the patient will experience cardiac dysrhythmias including atrial fibrillation c If shivering does not obscure the tracing a unique Osborn wave may occur d A hypothermic heart becomes susceptible to ventricular fibrillation at a CBT of F C AHA ACLS guidelines suggest attempting defibrillation once If V-fib persists treatment may include repeated attempts in conjunction with rewarming strategies iv Respiratory rate initially speeds up but then slows leading to a decreased minute volume a Tracheobronchial secretions increase b Bronchospasm may occur c Noncardiogenic pulmonary edema especially in the elderly may occur d At F C Profound hypoventilation Protective airway reflexes decline Oxygen consumption decreases by about half v Muscular system slows in response to cold a Initial reaction is shivering Generates heat but makes skilled movement more difficult b Shivering stops at around F C Cold muscles become progressively weaker and stiffer making it hard to save oneself vi Cold also affects the body s metabolism a Shivering depletes the body s glucose Leads to hypoglycemia Insulin levels fall and further glucose metabolism is impossible Body switches to fat metabolism Liver s metabolism of drugs slows b Causes the effects of drugs to last longer Management a General care aimed at preventing further heat loss and rewarming i Strip patient of wet clothes ii Insulate patient from further heat loss b Breathing patients with a pulse i Mild hypothermia cases F C a Treatment is passive rewarming including Removing wet clothing Drying the skin Moving patient to a warmed ambulance Using warm or space blankets b Depending on location and relative ease of transport may need to promote heat generation by Feeding the patient Giving warm fluids Having patient move around ii Moderate hypothermia cases F to F C to C a If patient has a perfusing rhythm treatment is active external rewarming involving the direct warming of the patient s skin including Heating blankets or radiant heat from hot packs placed in the groin neck and axillae Forced hot air Warmed IV fluids at temperatures from F to F technically considered active core rewarming b Administer a fluid bolus unless contraindicated to counter hypovolemia c Commercial warming devices with special blankets and a heated fan unit can warm up to F per hour much faster than warm blankets d Monitor for direct thermal tissue injury and hemodynamic changes because active external rewarming measures can cause afterdrop continued lowering of CBT Afterdrop more common in patients with chronic hyperthermia and hypothermia complicated by frostbite Caused by rewarming peripheral blood before core is warmed leading to peripheral vasodilation Inadequate fluid resuscitation may play a role e Paramedics working in regions where winter wilderness rescue operations are routine should carry specialized gear for hypothermia management Example Hydraulic sarong a thin double-layered blanket with network of tubing between two layers which is wrapped around the patient then hot water is pumped through the tubing Example Devices to help deliver heated humidified supplemental oxygen to aid in core rewarming iii Severe hypothermia cases Less than F C a Active core rewarming sequence is done in-hospital using Warm IV fluids Warm humid oxygen Body cavity lavage peritoneal or thoracic Extracorporeal rewarming Esophageal rewarming tubes b Cardiopulmonary bypass and continuous arteriovenous rewarming are most rapid methods but require specialized equipment and personnel c Continue rewarming until CBT is greater than F C Spontaneous circulation returns Resuscitative efforts cease c Patients with no pulse or not breathing i The BLS guidelines recommend CPR if no signs of life are present ii Rapidly do a rhythm identification iii One defibrillation attempt is prudent iv Follow local guidelines some argue that CPR should be withheld if there is an organized rhythm on the ECG v Establish IV access and infuse warm normal saline vi Attempt an advanced airway and ventilate with warm humid oxygen vii Commercial devices for both IV fluids and oxygen delivery are available viii The American Heart Association ACLS guidelines suggest considering administration of a vasopressor during cardiac arrest d Withholding and cessation of resuscitative efforts i Patients are generally considered dead if a There are obvious lethal traumatic injuries b They are so solidly frozen as to block the airway or chest compression efforts ii If submersion preceded the arrest resuscitation is unlikely a Possible exception if immersed in icy waters iii Trauma alcohol overdose and drug overdose can hamper resuscitation efforts a Factor these conditions into treatment decisions and seek medical control input iv Some providers feel that patients who appear dead after prolonged cold temperature exposures are not dead until warm and dead a Hypothermia effects may protect brain and organs if hypothermia develops quickly fact being used to treat some cardiac arrest patients b May be impossible to know if cardiac arrest or hypothermia came first in these situations prudent to attempt resuscitation V Pathophysiology Assessment and Management of Drowning A Pathophysiology fatal unintentional drowning in the United States in an average of people per day a Second leading cause of injury-related death among children younger than years b Numbers improved significantly in last century because of improved prevention and education c Numbers remain unchanged over past decade Definition of drowning adopted by the World Health Organization from First World Congress on Drowning process of experiencing respiratory impairment from submersion or immersion in liquid a Outcomes include i Death ii Morbidity iii Near morbidity b Terms no longer used to describe drowning include near-drowning and wet dry and secondary drowning The drowning continuum progresses from a Breath holding to b Laryngospasm to c Accumulation of carbon dioxide and inability to oxygenate the lungs d Subsequent respiratory and cardiac arrest from multiple organ failure from tissue hypoxia Patient can be resuscitated at any point along the continuum a The earlier the resuscitation the better the success rate Risk factors for drowning a Toddlers typically drown in bathtubs b School-age children pools c Teens lakes and rivers d Comorbidities seizure disorder medical physical handicaps may contribute in apparently safe environments bathtub Usually a predictable sequence starting with the victim unable to keep the face out of a liquid medium a Length of breath holding depends on i State of health and fitness ii Level of panic iii Water temperature duration shorter in colder water b As water enters the mouth and nose the victim may swallow large amounts of water during coughing and gasping i Theoretic difference between saltwater and freshwater drowning but information is not useful during resuscitation ii Both lead to pulmonary injuries but water composition does not generally affect electrolytes or blood volume c Water in the pharynx and laryngeal areas leads to laryngeal muscle spasms d Laryngospasm leads to asphyxia a combination of hypoxemia and hypercarbia and patient may lose consciousness i Hypoxemia stimulates body to shift from aerobic to anaerobic metabolism ii This leads to production of lactate and development of metabolic acidosis e Laryngospasm eventually ceases and water begins to enter the lungs f The initial hypoxic insult is believed to occur from apnea but as time progresses the lungs are injured directly in a process called acute lung injury i Lung tissue is directly damaged by complex biochemical processes ii Results in poor lung compliance and difficulty ventilating and oxygenating the victim iii Leads to hypoxic brain damage and deleterious effects of hypoxia iv May lead to dysrhythmias and cardiac arrest B Assessment Resuscitation of a drowning victim is generally the same as for any other patient in respiratory or cardiac arrest with a few exceptions a Must reach the victim i Best to leave it to those with training and experience in water rescue b Treatment steps follow ABCs i Establish airway ii Cervical spine precautions if necessary especially if a History of diving or water slide b Signs of injury c Alcohol intoxication C Management Continue rescue breathing until the patient is on land a Once on solid surface i Start supplementary oxygen ii Quickly determine if there is a pulse b In accordance with American Heart Association ACLS guidelines continue to treat according to ABC guidelines as opposed to CAB approach c Once two breaths are given start chest compressions and continue according to ACLS and Pediatric Advanced Life support guidelines i Establish IV access ii Administer indicated medications iii Perform cardiac monitoring iv Defibrillate any shockable rhythms a Only the chest needs to be dried prior to AED placement b Additional drying should be performed only when it will not delay further defibrillation attempts d Do not perform manual abdominal thrusts to remove water from lungs i May displace water from stomach into the lungs ii May increase risk of pneumonitis and subsequent lung infections e Suction to clear the airway f If BLS airway interventions fail place an early advanced airway Most drowning victims receiving rescue breathing or compressions will vomit a Remove vomit from mouth via i Suction ii Finger swipes iii Other devices b Consider placing on the side to avoid aspiration Some alveoli normally collapse during the expiratory phase of the respiratory cycle during normal spontaneous breathing a In drowning some positive pressure should be maintained at the end of exhalation to i Keep the alveoli open ii Drive any fluid accumulated in the alveoli back into the interstitium or capillaries b Positive end-expiratory pressure PEEP maintains some degree of positive pressure at the end of the expiratory phase i Indicated for intubated patients who are transported over long distances after submersion or who have conditions that produce significant shunt ii Several commercial devices are designed to allow PEEP via an endotracheal tube iii Portable ventilators usually have a PEEP setting If an ET tube has been inserted insert a nasogastric tube to decompress the stomach If pulse is absent implement ALS measures for cardiopulmonary arrest a Establish IV access b Administer epinephrine c Perform cardiac monitoring and defibrillation as necessary Patients rescued from submersion are likely to have bronchospasm from airway irritation a If wheezing administer a beta- adrenergic drug eg albuterol as if the patient were having an acute asthmatic attack b Corticosteroids however are associated with adverse outcomes in drowning victims Do not give up on a submersion patient especially if a child and the incident occurred in icy water a Successful resuscitation with complete neurologic recovery has occurred in cases of submersion for more than hour in icy water i Hypothermia decreases body s metabolic demand protecting the body and brain from effects of prolonged hypoxia ii Hypothermia is more often dangerous than protective so use the hypothermia algorithm to consider the effects of hypothermia on a drowning patient b Search for comorbidities including i Trauma ii Hypoglycemia iii Acute coronary syndrome iv Cerebrovascular accident v Alcohol or drug intoxication Major predictors of outcome include a Length of submersion b Response to field resuscitation c If patient is awake upon hospital arrival he or she will likely have a better outcome Paramedics may participate in injury prevention activities a Commenting on pool safety practices during a call b Formal participation in community education and prevention programs For a summarization of drowning management see Table D Postresuscitation complications Common complications occurring hours to days after submersion include a Adult respiratory distress syndrome b Hypoxic brain injury c Multiorgan failure d Sepsis syndrome Highlight importance of emergency department evaluation of submersion victims a Symptoms may be subtle or asymptomatic b Those who walk and talk then deteriorate are experiencing postsubmersion syndrome VI Pathophysiology Assessment and Management of Diving Injuries A There are million recreational scuba divers in the United States as well as commercial and military divers Americans receive scuba instruction every year Paramedics working in coastal and lakefront areas are likely to encounter diving casualties Four modes of diving a Scuba diving most popular named for the self-contained underwater breathing apparatus carried on the back b Breath-hold diving free diving no equipment used except possibly a snorkel c Surface-tended diving air piped to the diver through a tube on the surface d Saturation diving diver remains at depth for prolonged periods Divers are subject to increased ambient pressures found underwater a Injuries from physical effect of pressures on the body B General pathophysiology Physical principles of pressure effects Pressure a force per unit area a Can be expressed in different ways for example the weight of air at sea level can be expressed as i pounds per square inch psi ii mm Hg iii atmosphere absolute ATA used most commonly in diving medicine b Water is much denser than air so relatively small changes in depth produce large changes in pressure i For every feet of seawater fsw the pressure increases ATA ii Depth of the dive can be used to estimate the pressure to which the diver was exposed a At sea level the pressure is ATA b At fsw the pressure is ATA c At fsw the pressure is ATA iii Majority of scuba diving is done at depths between and fsw to ATA Liquids are not compressible their volumes do not change with pressure a The body and tissues are composed primarily of water so their volumes are not significantly affected by pressure changes in descent or ascent through water b Gas-filled organs are compressible and follow several physical laws c Boyle s law at a constant temperature the volume of a gas is inversely proportional to its pressure i If you double the pressure on a gas you halve its volume PV K a P pressure b V volume c K a constant ii The effect is the most extreme near the water s surface iii This law explains barotraumas that can occur in gas-filled spaces in the body a Lungs b GI tract c Sinuses d Parts of the ear d Dalton s law each gas in mixture exerts the same partial pressure that it would exert if it were alone in the same volume and that the total pressure of a mixture of gases is the sum of the partial pressures of all gasses in the mixture i Fresh air Ptotal PO PCO PN ii When total pressure increases partial pressure of each gas increases proportionally iii Helps explain a Nitrogen narcosis b Oxygen toxicity c Contamination dangers in pressurized breathing systems iv Because the relative percentage of each gas remains constant at different pressures it shows why pulse oximetry readings in divers with nitrogen narcosis remain unaffected e Henry s law the amount of gas dissolved in a liquid is directly proportional to the partial pressure of the gas above the liquid P kC where i P partial pressure of gas above the liquid ii k a constant iii C concentration of gas in the liquid iv Classic example when a sealed bottle that has dissolved carbon dioxide is opened a Opening the bottle allows carbon dioxide to escape which lowers the partial pressure of the gas b This forces carbon dioxide concentration in the soda to decrease c The remaining carbon dioxide escapes as bubble v Henry s law explains decompression sickness f Fresh air nitrogen and oxygen i Nitrogen an inert fat-soluble gas g Nitrogen in the body follows the laws above i Boyle s law the volume of nitrogen decreases as pressure increases during descent and its volume increases as pressure decreases during ascent leading to barotrauma ii Dalton s law partial pressure of nitrogen increases as total pressure increases a At high enough partial pressure nitrogen becomes an anesthetic leading to nitrogen narcosis at depths nearing fws iii Henry s law as pressure decreases during ascent nitrogen comes out of the solution in the blood and may cause bubbles in tissues leading to decompression sickness the bends h Oxygen also follows these laws i Can cause oxygen toxicity at depths the same way it causes nitrogen narcosis ii Because oxygen is metabolized by the body it does not contribute to decompression sickness Commercial divers use a decompression schedule to allow gases time to equilibrate a Recreational divers usually have a no-decompression limit to keep from decompressing on surfacing b Using enriched Nitrox gas a gas with lower nitrogen and higher oxygen decreases the risk of nitrogen narcosis and decompression illness DCI but increases risk of oxygen toxicity c Dive tables and dive computers give guidelines for taking decompression stops during the dive but are not foolproof C General assessment Diving history Obtain as many details as possible about the dive and onset of patient s symptoms a When did symptoms start during ascent or descent i Decompression sickness usually manifests within first hour to hours of surfacing ii If patient flew after diving decompression sickness may start within hours after the dive iii If symptoms occur within minutes of surfacing especially if accompanied by loss of consciousness consider an air embolism b Type of diving and equipment c Type of tank compressed air or Nitrox system d Diving site and water temperature e Number of dives in the last hours and i Depth ii Bottom time iii Surface interval f Dive computer used g Safety stops used h Any attempts at in-water decompression i Any dive complications j What were predive and postdive activities D Injuries at depth Pathophysiology a Nitrogen narcosis altered mental status caused by breathing compressed nitrogen-containing air at depth i Nitrogen dilutes concentration of oxygen ii Nitrogen also makes up of compressed air Dalton s law iii When pressure increases partial pressure of nitrogen also increases iv At partial pressure of about ATA at about ft nitrogen begins to have anesthetic properties v Effects become more pronounced at fsw Assessment a Signs and symptoms include i Euphoric feeling ii Inappropriate behavior at depth including lack of concern for safety iii Tingling of lips gums and legs iv Diver may panic and spit out the regulator or surface too quickly v Divers are better able to tolerate breathing compressed air as experience increases Management a Must lower nitrogen partial pressure through i Controlled ascent ii Use of a mixed gas for diving with a decreased nitrogen percentage E Injuries during descent and ascent Barotrauma a Pathophysiology i Barotrauma squeeze the major problem divers encounter ii Results from pressure imbalance between gas-filled spaces in the body and the external atmosphere iii Two different mechanisms a Compression of gases within body spaces during descent b Expansion of gases within those spaces during ascent iv Can affect any gas-filled space in the body including a Sinuses b Inner and middle ears c Teeth v Scuba divers general protected from barotraumas by breathing compressed air matching the pressure from the surrounding area a As long as the air-filled cavities can equilibrate freely they will not implode b This is not true if there is an obstruction sinus or ear infection vi As the diver ascends and the ambient pressure around him or her decreases the gases in the body s air-filled spaces expand a Commonly affects ears and sinuses b Example If a diver uses decongestants before a dive and the medication wears off before ascent the increased mucosal swelling keeps air trapped in the sinuses and ears creating increasing pressure that cannot equalize during ascent reverse squeeze b Assessment and management i The pressure in the middle ear cannot be equalized with the pressure of the outside water if a There is a blockage in eustachian tube b The diver cannot equalize ear pressures with a Valsalva maneuver ii A middle ear squeeze syndrome may develop with severe ear pain iii If the tympanic membrane ruptures the patient may experience especially in colder waters a Nausea b Vomiting c Vertigo iv If the diver is at depths this may cause a Panic b Rapid ascent c Problems associated with ascent v Treatment includes a Loose dressing for ear bleeding b IV antiemetics or sedatives as necessary vi Some symptoms may be a sign of decompression sickness such as a Hearing loss b Vertigo Pulmonary overpressurization syndrome POPS a Pathophysiology i If divers fail to exhale during an ascent pressure in the lungs increases a Lung volume of scuba divers that inhaled to their total lung capacity at fsw ATA doubles by the time they reach the surface if holding their breath during ascent b Likely to occur in emergency ascent when divers panic and hold their breath ii POPS also known as burst lung is one of worst forms of barotraumas and can cause a Pneumothorax b Mediastinal and subcutaneous emphysema c Alveolar hemorrhage d Lethal arterial gas embolism AGE iii Because relative pressure and volume changes are greatest near the surface of the water a small overpressurization eg holding the breath for the last ft of ascent can rupture alveoli a Because of this diving students are trained to exhale constantly during ascent to vent air from the lungs iv Those with COPD and asthma have a slightly higher risk because of their already altered air movement dynamics b Assessment and management i Signs and symptoms depend on where the air escaping from alveoli rupture ends up ii Most often leaks into the mediastinum and beneath the skin causing mediastinal and subcutaneous emphysema with symptoms including a Full sensation in the throat b Pain on swallowing odynophagia c Dyspnea d Substernal chest pain e Hoarseness or brassy quality to the voice iii Physical examination may show a Palpable subcutaneous air above the clavicles b Crunching noise synchronous with the heartbeat audible by auscultation Hamman s crunch iv Less common result may be pneumothorax with symptoms including a Unequal breath sounds b Low pulse oximetry values c Hyperresonance on affected side of chest v Prehospital treatment depends on whether the patient has an arterial gas embolism see next section a If patient has only pneumomediastinum and subcutaneous emphysema will likely be managed symptomatically in the hospital b Pneumothorax may need needle decompression or a chest tube vi In the field provide oxygen by nonrebreathing mask a Be careful not to give PEEP to a patient with POPS b PEEP increases oxygen s partial pressure which may decrease bubble size and speed up off-gassing Arterial gas embolism AGE a Pathophysiology i AGE the most dangerous possible consequence of POPS which is second only to drowning as cause of death to divers ii Air bubbles from ruptured alveoli enter pulmonary capillaries and coalesce into larger bubbles as they travel through the pulmonary veins back to the left side of the heart iii Bubbles may enter the coronary arteries from the left ventricle and produce the effects of acute myocardial infarction including cardiac arrest iv The vast majority rise to the head causing strokelike symptoms within the cerebral circulation b Assessment i Clinical picture is dramatic with symptoms involving most cerebral functions and appearing within seconds or minutes usually within minutes after surfacing ii History of panic or uncontrolled ascent but can occur in shallow water iii Patient can experience a Weakness or paralysis of one or more of the extremities b Seizure activity c Unresponsiveness d Other neurologic symptoms including Paresthesias Visual disturbances Deafness Changes in mental status c Management i If AGE is suspected transport to hyperbaric chamber facility as soon as possible ii Treatment of suspected AGE includes a Ensuring adequate airway Intubating with advanced airway in an unresponsive patient Filling air balloons with saline instead of air to allow for hyperbaric therapy b Administering supplemental oxygen by nonrebreathing mask c Transporting in supine position Avoid Trendelenburg position Monitor treat for hypothermia if necessary d Transporting by ground is preferred to air because of cabin pressure Use most expeditious form of transportation to a center with hyperbaric oxygen capabilities e Establishing IV access then administering normal saline f Monitoring cardiac rhythm and treating any dysrhythmias g Having medications ready for immediate use Sedatives for seizures lorazepam midazolam diazepam Dopamine infusion for hypotension mcg kg min h Using local protocols for direct referral to hyperbaric chamber facility i Following local protocols for the controversial use of lidocaine and steroids Decompression sickness DCS a Pathophysiology i A broad range of signs and symptoms caused by nitrogen bubbles in blood and tissue coming out of solution during ascent ii Bubbles do damage by a Interfering mechanically with tissue perfusion b Triggering chemical changes within the body iii Multisystem disorder can potentially affect almost every organ in the body iv Nitrogen is highly lipid soluble making the CNS and spinal cord more susceptible to DCS v Per Henry s law increasing amounts of nitrogen and oxygen dissolve in the blood as a diver descends a This is carried to tissues where oxygen is metabolized but nitrogen remains b During the ascent ambient pressure decreases and the reverse happens Nitrogen starts to diffuse out of the tissues If ascent is slow enough the amount of nitrogen will equilibrate with that in the alveoli and escape with each breath If the ascent is more rapid than nitrogen can be removed the diver s tissues will begin to bubble Effect worsens with multiple dives in a short time without allowing for nitrogen off-loading after each dive vi Other risk factors for DCS include a Obesity b Dehydration c Fatigue d Flying within to hours of diving vii Diving itself can lead to dehydration especially in tropical climates while breathing dry air viii Risk of severe neurologic DCS is a patent foramen ovale present in about one third of the general population a Congenital defect in which the foramen ovale between the atria fails to close at birth b May allow nitrogen bubbles to travel from pulmonary circulation directly into systemic circulation c Leads to increased damage of the CNS and proximal spinal cord b Assessment i Type I mild forms that involve only the a Skin b Lymphatic system c Musculoskeletal system ii Symptoms include a Joint pain most common causes the patient to bend over in pain b Mottled and pruritic skin c Fatigue and weakness d Rarely lymph dysfunction that can lead to edema iii Type II much more dangerous with symptoms in all other organs including a Pulmonary systems b Cardiovascular systems c Nervous systems iv DCS can be described by specific system affected and precise symptoms c Management i May not be able to distinguish between DCS and AGE in the field especially when neurologic symptoms are present a DCI sometimes used for both DCS and AGE Symptoms from air embolism usually reflect cerebral dysfunction DCS more likely to involve spinal cord injury Loss of consciousness likely to be AGE ii Management in either case is the same a Administer oxygen b Manage acute problems such as dysrhythmias and seizures c Transport to hyperbaric facility even if symptoms appear to resolve iii Treatment of DCS summarized as a Ensure adequate airway b Administer supplemental oxygen by nonrebreathing mask c Normal saline through IV line administer fluids at ordered rate In long-range transport of catheterized patient adjust to produce urine output of to mL kg h d Do not use nitrous oxide oxygen for analgesia e Per protocol arrange for transport to a hyperbaric facility If unsure which is the closest facility contact the Divers Alert Network DAN at - d Hyperbaric oxygen therapy i Therapy involves intermittent inhalation of pure oxygen under pressure greater than ATA a Mechanically reduces bubble size b Reduces nitrogen content c Increases oxygen delivery to ischemic tissues ii Treatment pressures and times from established tables such as US Navy Treatment Tables iii Indicated in patients with a AGE b DCS c Carbon monoxide poisoning d Other subacute or chronic medical conditions e Routine decompression of industrial divers iv Risks include a Possible conversion of a pneumothorax into a tension pneumothorax if there is no chest tube b Possible seizures from oxygen toxicity c Possible barotraumas via same mechanism as diving v Due to potential for barotraumas or seizures care is recommended in patients who a Are pregnant b Have lung disease such as asthma or COPD c Have a fever d Have seizure disorders vi In patients with possible AGE or DCS benefits may outweigh the risks F Other gas-related problems Pathophysiology a Most recreational divers breathe compressed air i Tanks holding various mixtures of nitrogen and oxygen allow divers to remain underwater longer ii Breathing less nitrogen means divers less likely to develop DCS iii Makes divers more prone to oxygen toxicity a CNS emergency iv Air with an oxygen concentration of can cause toxicity at a depth of ft Assessment a Signs and symptoms include i Dizziness ii Lack of coordination iii Confusion iv Twitching or paresthesia symptoms v Underwater seizures Management a Evacuation from the water involves i A controlled ascent to the surface ii Ensuring the diver maintains an airway and has access to air during ascent b DCS may be a concern after an emergent ascent but AGE risk does not increase when a seizing or postictal patient is brought to the surface c Rarely a tank may be filled with contaminated air especially if the compressor malfunctions i Carbon monoxide and carbon dioxide affect the diver early in the dive a Characteristic that helps distinguish the condition from DCS d Once in the boat or back on shore treatment includes i oxygen ii Supportive therapy iii Hyperbaric oxygen therapy in pregnant patients or those with significant exposure decreases long-term neurologic consequences of carbon monoxide poisoning G Shallow water blackout Pathophysiology a Shallow water blackout can be seen in any part of the country b Frequently occurs in adolescent boys competing in remaining the longest underwater i May hyperventilate just before diving under the surface to extend endurance a Decreases PaCO and causes cerebral vasoconstriction b As the swimmer descends the PaO increases c Because PaCO is relatively low the respiratory drive is suppressed d The diver can remain underwater longer than normal but oxygen continues to be removed from the alveoli e The cerebral function is maintained at depth by the increased PaO f On surfacing Ambient pressure rapidly decreases PaO plummets Hypoxemia and cerebral vasoconstriction together causes blackout just before reaching surface Assessment and management a Treat as for any other case of drowning b When patient regains consciousness explain the seriousness of the injury c Explain to the victim and bystanders that hyperventilation before a breath-hold dive can result in death H Getting help for diving injuries Resource for emergency medical personnel dealing with underwater diving accidents the DAN a -hour emergency consultation service a - b Calls received at DAN headquarters at Duke University Medical Center Durham North Carolina c Caller connected with a physician experienced in diving medicine who can i Assist with diagnosis ii Provide advice for early accident management iii Supervise referral to appropriate recompression chamber if necessary d The DAN produces training and continuing medical education resources found on their website VII Pathophysiology Assessment and Management of Altitude Illness A Altitude terrestrial elevation above m ft The level where the physiologic changes begin due to the hypobaric hypoxia the body is exposed to a Altitude illness low partial pressure of oxygen leads to hypoxia i Dalton s law see section on diving amount of gas available for use determined by partial pressure driving the gas ii As ascends in elevation atmospheric pressure decreases iii Partial pressure of oxygen decreases hypobaric iv Results in decreasing amount of oxygen available v Where partial pressure of oxygen decreases with increasing altitude it remains a constant of atmospheric gasses a Example Partial pressure of arterial oxygen PaO is mm HG at sea level but mm Hg in Denver elevation ft Barometric pressure varies according to a How far from the equator lower the farther from the equator b Season typically lower in winter c Local changes in barometric pressures which can alter relative altitude by to ft B Pathophysiology Altitude illnesses illnesses caused by effects of hypobaric hypoxia on the CNS and pulmonary system as a result of unacclimatized people ascending to higher altitudes Types of illness include a Acute mountain sickness AMS most common b High-altitude cerebral edema HACE c High-altitude pulmonary edema HAPE Typically occurs in those who rapidly ascend to heights about ft but can occur at altitudes as low as ft a Symptoms usually occur within to hours b Incidence directly related to both how high people ascend and how quickly they arrive at that elevation i If ascent is slow enough the body will acclimate to altitude without developing illness ii Altitude illness develops when ascent is higher or faster than the body can acclimatize to the altitude a Studies show that skiers who go from sea level to higher elevations and sleep at around feet have a chance of developing AMS b Climbers of Mt Rainier typically going from sea level to ft in as little as to hours have a chance of developing AMS The body adjusts or acclimatizes to altitude by defending the amount of oxygen available for delivery to the tissues a First response to hypobaric hypoxia is hyperventilation allowing for blowing off of carbon dioxide in exchange for holding onto oxygen b Quickly leads to respiratory alkalosis for which the body must compensate in order to continue to hyperventilate c This is done by kidneys secreting bicarbonate in the urine d Causes a compensatory metabolic acidosis and the body can continue to hyperventilate e Other changes that take several days to a week also occur i Increased production of red cells ii Changes at cellular level to improve oxygen transfer Hypoxia is the main problem behind pathophysiologic responses in altitude illness but the exact mechanism is poorly understood a Hypoxia initiates a complex series of reactions that cause overperfusion to the brain and lungs which increases i Capillary pressures ii Leakage iii Cerebral and pulmonary edema b HAPE results from marked vasospasm of the pulmonary arteries i Results in high pressure driving fluid from pulmonary vasculature into the lungs causing pulmonary edema ii Does not result from volume overload state iii Treatment for HAPE is different than standard pulmonary edema a Nitroglycerin and furosemide are not used for HAPE Risk factors for altitude illness a Risk factors include i Prior history of AMS a Slow ascent and or use of prophylactic medicines ii Normal residence below ft iii Obesity iv Rapid or high ascents v Physical fitness not a factor vi Age not a factor although some suggest that the elderly are less likely to develop altitude illness C Assessment Altitude illness develops when people ascend to an elevation higher or faster than their body can acclimatize a A spectrum of illnesses ranging from i Acute mountain sickness AMS clinical diagnosis with no specific physical findings with symptoms including a Headache often throbbing worse over the temporal or occipital areas and exacerbated by the Valsalva maneuver plus at least one of the following b Fatigue or weakness c Gastrointestinal symptoms d Dizziness or light-headedness e Difficulty sleeping ii High-altitude pulmonary edema HAPE life-threatening with at least two of the following symptoms a Dyspnea at rest b Cough c Weakness or decreased exercise performance d Chest tightness or congestion e Two of the following signs Central cyanosis Audible rales or wheezing in at least one lung field Tachypnea Tachycardia iii High-altitude cerebral edema HACE life-threatening a The presence of a change in mental status or ataxia in a person with AMS b The presence of mental status changes and ataxia in a person without AMS Other conditions can mimic AMS a Signs of possible other causes i Symptoms that develop or more days after being at higher elevations ii Lack of headache iii Failure of descent to improve signs or symptoms D Management Management of all altitude illness includes a Oxygen b Descent c Evacuation d Various treatments specific to illness Prevention by a Acclimatization b Use of acetazolamide for those most likely to be susceptible i A carbonic anhydrase inhibitor causing kidneys to secrete bicarbonate ii Causes metabolic acidosis which causes the body to hyperventilate and creating a respiratory alkalosis iii Helps the body acclimatize by causing needed physiologic changes iv Decreases cerebrospinal fluid production through decreasing minimal cerebral edema v Side effects include a Diffuse and migratory paresthesias b Mild diuresis c All carbonated beverages taste bad because it hydrolyzes carbon dioxide on the tongue vi A sulfa-based drug so it should be avoided by those with sulfa allergies vii The only drug with FDA approval for prevention and treatment of altitude illness Standard for slow ascent ascend to sleeping altitude only meters per day after elevation of meters Treatment of AMS can be both symptomatic and physiologic a Acetaminophen or aspirin for headaches b Antiemetics for nausea prochlorperazine ondansetron c Acetazolamide for AMS and to help the body acclimatize to altitude d Oxygen if available e AMS patients do not necessarily need to descend although it would rapidly improve AMS but should not ascend further until symptoms resolve HAPE treatment includes a Descent should descend immediately b Oxygen c Medication as adjunctive treatment is limited to patients where i Descent and oxygen are not easily available ii Descent and oxygen do not rapidly help the patient d If significant descent and oxygen fail to improve the illness consider a different cause for the pulmonary symptoms e Medications to treat and prevent HAPE are predominantly pulmonary artery vasodilators i Nifedipine is the gold standard for prevention and adjunctive treatment ii Phosphodiesterase inhibitors sildenafil and tadalafil are used for treatment of pulmonary hypertension and are also used for prevention and treatment of HAPE iii Albuterol should be as effective as salmeterol iv Dexamethasone has been shown to be effective in prevention but is not routinely used v Nitrates and furosemide should not be used because HAPE is not caused by fluid overload HACE treatment includes a Oxygen b Mandatory descent c Use of dexamethasone i Should be given as soon as possible ii Dose of mg given by any accessible route followed by mg every hours during descent evacuation Portable hyperbaric chambers a Useful when descent cannot be carried out b Available as i Gamow bag ii The PAC iii Certec bag c Patient is placed inside bags and pressurized air is pumped in by foot or electric pump i The increased pressure around the patient provides the equivalent of descent of several hundred to several thousand feet d Even more effective if patient is given oxygen while inside the bag e Bags are cramped claustrophobic and make it difficult to treat the patient once inside VIII Pathophysiology Assessment and Management of Lightning Strike A Pathophysiology Lightning strike is the leading cause of environmental death in the United States a Causes to deaths a year in the United States i Florida is the leading state for lightning strikes b Morbidity is even higher three to five times more people are struck than are killed c Different than industrial electrical injuries i Not AC or DC current ii Massive unidirectional flow of electrons with voltages in the millions up to million or more volt range iii Duration is miniscule compared to electrical current only one thousandth or ten thousandth of a second iv Energy typically flows over victim rather than being transferred into them The most common lightning injury comes from a side splash injury where lightning hits an object then spreads from that point The lightning bolt s energy may act as a giant depolarizing charge to the entire body a Can cause asystole and respiratory arrest via i Diaphragm depolarization ii Brainstem-induced central apnea b After depolarization i The heart will usually spontaneously resume a sinus rhythm ii The body s respiratory effort does not spontaneously restart iii If the patient remains apneic the heart will go into a secondary hypoxic arrest c If CPR is begun promptly most will survive d Morbidity remains high with as many as of people struck by lightning having long-term complications i Clinically significant burns not usually seen unless patient a Had metal objects on that heated up causing thermal burns b Was wet and the water converted to steam causing steam burns ii May have a Lichtenberg figure not an actual burn but caused by electron shower on the skin iii Victims may have been thrown by the strike evaluate for trauma B Assessment and management Initial approach and triage should be reverse triage a If signs of life will likely recover so attend to those who appear dead first b Lightning strike induces cardiac and respiratory arrest i Heart often restarts spontaneously but respirations will not ii Causes the victim to become hypoxic preventing the heart from recovering iii CPR rescue breathing may prevent hypoxia and the heart may restart All victims of lightning strike should be evaluated at a medical facility IX Pathophysiology Assessment and Management of Envenomation Bites and Stings A There are thousands of creatures that produce and secrete venom in the continental United States Most do not have an effective method of injecting or spreading venom to humans The commonly encountered venomous creatures include a Insects of the hymenoptera species bees wasps hornets and fire ants b Snakes of the Elapidae and Crotalid species c Spiders of the Latrodectus mactans black widow Loxosceles reclusa brown recluse and Tegenaria agrestis hobo variety d Scorpions of the Centruroides species Anaphylactic reaction is the most frequent cause of mortality from all bites and envenomations a Can occur with exposure to any venom but most common with hymenoptera envenomation with the most deaths related to hymenoptera bites b Treatment of anaphylaxis due to bites and envenomation is the same as treatment from any other cause of anaphylaxis as outlined in the chapter Immunologic Emergencies Treatment for all envenomations that do not include anaphylaxis is a ABCs management b Transport c Antivenins not commonly used in prehospital settings in the United States d Attainment of vascular access with all transported patients e Check that scene is safe from venomous creatures i Patient or friend may have captured the creature for identification ii Be careful handling some creatures may cause envenomation even after death B Hymenoptera Pathophysiology a The most common cause of envenomation-related deaths is the hymenoptera order i European honey bees Aphidae ii Yellow jackets wasps hornets Vespidae iii Fire ants Formicidae b An estimated more than million hymenoptera stings annually in the United States i require treatment for anaphylaxis c Degree of cross-reaction in the venom of all three subfamilies i A person who becomes anaphylactic after a bee sting could also become anaphylactic after a fire ant bite d With honeybee stings the stinger venom sac is pulled from the bee and remains attached to the skin Assessment a Venom of hymenoptera is a mixture of proteins that cause a local reaction with symptoms including i Erythema ii Swelling iii Pruritus b Melittin a protein in the venom that causes the immediate pain from a sting i Works through direct effect on skin pain receptors c The more severe hymenoptera sting reactions are immunoglobulin-E mediated d In a quarter of stings a local reaction develops i Reaction can be extensive over the day or two after the sting ii Exceedingly rare for an infection to develop e Anaphylaxis occurs rapidly i Typically within minutes ii Within minutes for of the time iii Rarely takes up to hours Management a If there is no history of allergic reaction to bee stings and no systemic reaction transport is usually unnecessary i Advise patient of anaphylaxis warning signs and the urgency of calling - - ii Tell the patient to have a physician check the wound if it does not improve within hours iii Infection is likely after fire ant stings a Typically produce small pustules at the site about hours after the sting b When these break open often from scratching the area is vulnerable to secondary infection b Treatment of hymenoptera sting focuses on pain and infection risk minimization i Determine if stinger and venom sac are still attached to the skin a Remove the stinger as rapidly as possible using the most readily available technique b Rapid removal has been found to be more important than gently scraping it from the wound ii After stinger removal clean the wound thoroughly with soap and water or antiseptic solution iii Treat local reactions with cool compresses and elevation iv Antihistamines can be used for symptomatic treatment a Can be given orally or sprayed topically onto the sting site b Topical hydrocortisone can be used c Additional for fire ant stings i Move patient and crew away from the site ii Brush off ants iii Provide supportive care C Snake bites Pathophysiology a Two families of snakes in the United States of concern for envenomation i The Viperidae causes of all bites ii The Elapidae b Approximately snake bites in the United States every year c Only present for actual treatment d Morbidity can be significant but fatalities are rare i Only four to five fatalities per year e Bites most commonly seen in the southeastern United States i Texas has greatest number of recorded snake fatalities f Pit viper venom contains both hemolytic and proteolytic enzymes that cause extensive local tissue damage and systemic effects causing i Soft-tissue swelling and necrosis ii Local then systemic bleeding iii Clotting problems g Coral snakes have potent neurotoxic venom causing i Paresthesias ii Fasciculations iii Weakness iv Respiratory difficulty v Strokelike symptoms vi Little direct local tissue injury h Crotalids Pit vipers i Vipiridae subfamily Crotalidae ii Cause of greatest number of bites and morbidity iii Name derived from heat-sensing pits located between the eye and the nostril iv Elliptical eyes and a single row of subcaudal plates v Commonly encountered pit vipers include a Rattlesnakes b Cottonmouths c Copperheads vi Eastern and western diamondbacks cause the greatest morbidity and mortality i Elapids Coral snakes i Three main coral snakes in North America a Eastern coral snake b Texas coral snake c Sonoran coral snake ii Difficult for coral snakes to inject venom because fangs are so small iii Snake must stay attached to cause envenomation so handlers are most typically bitten iv Envenomation causes few local symptoms v May take several hours before systemic symptoms develop a Any patient with possible elapid bite should be transported vi In the continental United States the elapid can be described by its banding pattern a Red on yellow kill a fellow red on black venom lack Assessment a Be certain scene is safe and that the snake is not still a threat by confirming it is either i Dead ii Trapped iii Gone b Note time of bite important to know amount of time that passes from time of bite to time of care c If possible determine type of snake i Remember that dead snakes can still bite d Venoms i Crotalid venom a mixture of enzymes that promote tissue destruction through a Proteolysis b Hemolysis c Thrombogenesis d Some degree of neurologic and cardiac toxicity in higher doses ii Venom components cause signs and symptoms of toxicity iii Severity determined by amount of venom injected and location iv Up to of bites do not have venom injected dry bites v Severity of bite difficult to determine immediately transport all patients vi Elapid venom primarily a neurotoxin that leads to respiratory failure and death a Mojave rattlesnake an exception to typical separation of crotalids and elapids It is a crotalid but its venom also has a potent neurotoxin Its bites are a combination of crotalid and elapid envenomation e Crotalid bite symptoms i Swelling and pain at bite site ii Bleeding at the site a Fang marks with no bleeding indicate a dry bite but patients should still be evaluated at an emergency department iii As severity increases swelling progresses up the extremity iv Systemic symptoms occur a Abnormal taste in the mouth metallic b Weakness c Dizziness d Altered mental status e Unconsciousness f Diaphoresis g Tachycardia h Nausea and vomiting i Coagulopathies develop leading to spontaneous bleeding j Shock and cardiovascular collapse v Degree of envenomation determined by degree of swelling and presence of systemic symptoms a Mild envenomation minimal local swelling only with no systemic symptoms b Moderate envenomation swelling extending up the extremity presence of systemic symptoms Coagulation abnormalities are absent or minimal No significant bleeding c Severe envenomation extensive soft-tissue swelling and severe systemic effects and bleeding up to and including Intravascular coagulation Shock Coagulation abnormalities Management a Treatment is provision of antivenin i Protocols should be in place for appropriate triage to facilities with antivenin b Monitor ABCs i Place an IV line and provide oxygen ii Keep patient calm supine and motionless c Clean wound with available antimicrobials d If protocols allow draw blood for hospital use because blood obtained later may produce unreliable coagulation studies e Immobilize the involved extremity in a neutral position below the level of heart f Do not apply excessive constriction g Remove rings and other constricting jewelry because swelling may occur rapidly h Begin immediate transport to a facility where the patient can receive antivenin D Spider bites Pathophysiology a An estimated species of spiders worldwide b Most are carnivores and can bite normally using their venom to subdue prey c Three important spider species in the United States i Black widow ii Brown recluse iii Hobo spiders d Only female black widows are dangerous to humans i Named for the practice of devouring her mate ii Identifying features of female black widow a Glossy black with -inch oval body b Leg span of about inch c Characteristic orange or reddish hourglass mark on the abdomen iii Found throughout the United States especially in the Southeast iv Most of the to bites occurring in the United States happen between April and October v Makes its home in sheds basements garages woodpiles and similar areas vi Likes to live in outhouses so bites may occur on sensitive areas vii Most bites are on the hands or forearms e Brown recluse spider also known as the fiddleback spider is found from the southern Midwest to Texas and across the Southeast i Not found in the far West unless accidently transported there f Hobo spiders member of the Tegenara species found in the Northwest i Bite clinically similar to brown recluse ii Brown recluse is not aggressive and bites only when accidently encountered iii Hobo spider is slightly more aggressive Assessment a A history of spider bite cannot always be confirmed b Patient may report i Sudden sharp prick followed by cramping ii Numbing pain that begins at the bite area and gradually spreads iii Classic sign of black widow spider bite extreme restlessness c Black widows are one of most venomous North American spiders i Predominantly a neurotoxin that causes local pain and swelling then muscle spasm and paralysis a Triggers multiple neurotransmitters from presynaptic nerves ii Local pain with rapid onset within to minutes of the bite iii Local muscle spasm and localized diaphoresis may occur iv Diffuse and more muscular spasms may follow a Typically involve thigh and shoulder girdle and later the abdomen b Abdominal spasms can be confused with an acute abdomen and may cause a boardlike appearance v May have nausea and vomiting vi If diaphragm is affected respiratory difficulty may develop d Bites of the brown recluse are usually painless i Most have few or no subsequent symptoms either local or systemic ii A small percentage will have local and or systemic symptoms iii A small subset of those will have more pronounced symptoms and will develop loxoscelism a A potentially fatal condition that Begins with a painful inflamed vesicle May progress to a gangrenous sloughing of skin b Most commonly presents cutaneously with pruritus and pain developing to hours later c Symptoms worsen over the next day to day and a half d Followed by development of a necrotic lesion that can take a month or more to heal and may require skin grafting iv Systemic symptoms much rarer and include a Nausea b Vomiting c Fever v Rarely hemolysis and coagulopathies have led to death Management a Treatment for black widow spider bite includes i Intermittent use of ice ii Antimicrobial cleansing of the wound iii Providing pain and muscle spasm relief iv Prompt transport b Further treatment includes i Standard ABCs and monitoring ii Obtaining IV access and applying oxygen iii Narcotics and muscle relaxants sedatives a Severe muscle spasms treated with benzodiazepines diazepam or lorazepam b Narcotics for pain relief iv Antivenin is available but its use is typically reserved for young and old with severe envenomation a High incidence of allergic reactions in all equine-derived antivenin c Treatment for brown recluse and hobo spider bites is supportive i Antivenin is not routinely available ii Dapsone had been recommended for brown recluse bites but has recently been shown to be of no benefit iii Previous treatments also shown not to be of any benefit include a Electric shock b Steroids c Hyperbaric oxygen d Colchicine e Antihistamines f Early excision E Scorpion stings Pathophysiology a Approximately scorpion species in continental United States i Only the bark scorpion Centruroides species a potential threat to humans b scorpion stings in the United States i Most result in only a painful local reaction c Bark scorpion found along Mexican border of Arizona and California with habitats up to parts of Nevada New Mexico and Utah i Not aggressive but crawls into places like shoes ii Nocturnal iii Climbs walls where it can be accidently encountered d Venom located in glands of the stinger i Stabs its prey after grasping it with the claws ii Venom a mixture of many toxins that affect nearly every body system Assessment a Local sting symptoms similar to hymenoptera stings with i Erythema ii Pruritis iii Urticaria iv Sharp burning pain v Paresthesia a Reported as feeling like a strong electric shock b Local effects begin within minutes and last for several hours c Local tissue necrosis is uncommon d Symptoms do not go past local sting area in vast majority of cases e A sting from the more neurotoxic bark scorpion causes few local symptoms i Systemic symptoms begin within minutes ii Peak at to hours iii Resolve within to hours f Autonomic excitation from neurotoxins is the most predominant effect i Toxin causes nerve sodium channels to remain open ii Allows continuous activation of nerves of the three nerve systems a Sympathetic b Parasympathetic c Somatic iii Symptoms due to diffuse neuronal excitation g Symptoms depend on which portion of the nervous system is most in overdrive i Sympathetic stimulation symptoms include a Tachycardia b Palpitations c Tachypnea d Hypertension e Dry mouth f Elevated temperature g Sympathetically-induced pulmonary edema ii Parasympathetic stimulation symptoms include a Bradycardia b Hypotension c Lacrimation d Salivation e Urination f Priapism g Defecation h Cranial nerve findings such as Nystagmus Wandering eye movements Tongue fasciculations Dysphagia iii Somatic involvement symptoms include a Muscle contractions b Myoclonic jerking c Fasciculations iv Typical victim has a Nystagmus b Hypersalivation c Dysphagia d Mydriasis e Restlessness Management a Prehospital treatment is primarily i ABC maintenance ii Monitoring iii Transport iv Intubation if necessary v Insertion of IV line with volumes given as required for blood pressure maintenance vi Ice pack application as necessary for local pain and swelling vii Immobilization of extremity viii Application of a constricting band just above the wound site not tight enough to occlude pulse may reduce lymph flow and spread of venom follow local protocols ix If seizures occur treat with benzodiazepines per local protocol and transport b A patient with a bark scorpion sting may show signs of severe toxicity i Anascorp a scorpion antivenin was approved by the FDA in ii An ovine-derived antivenin it can rapidly decrease morbidity in cases of severe toxicity c In-hospital treatment includes supportive care for ABCs i Use of alpha and beta blockers ii Atropine to control excessive nervous system symptoms iii Vasoactive drugs to control neurologic overstimulation iv Bark scorpion antivenin if available F Tick bites Pathophysiology a Blood-sucking arthropods found around the world often in rural wooded areas b Normally bites are not a medical emergency but a concern because they serve as disease vectors transmitting i Bacteria ii Viruses iii Protozoa c Linked to a variety of serious illnesses i Febrile illness ii Lyme disease associated with a bull s-eye rash around bite location iii Rocky Mountain spotted fever iv Tularemia d Rarely a bit on the back of the head neck or spine may cause a potentially life-threatening tick paralysis that cannot be reversed until the tick is removed i Presentation mirrors Guillain-Barr syndrome ii Consider if patient has unexplained weakness after recently being in the woods Assessment and management a Principle treatment is careful removal of the tick i Ticks attach themselves using mouth parts and a cementlike adhesive ii The mouth parts may remain embedded if the tick is pulled away from the skin iii To remove the tick a Put on gloves b Use a curved forceps to grasp the tick by the head as close to the patient s skin as possible c Pull straight upward using steady gentle traction and even pressure d Do not twist jerk squeeze or crush its body e Dispose of it in a container of alcohol b Once the tick is removed i Wash the bite with soap and water ii Depending on local protocol there may be no reason to transport the patient if asymptomatic iii Advise the patient to see a physician iv If you suspect tick paralysis or Lyme disease transport the patient X Summary A Environmental emergencies are medical conditions caused or worsened by weather terrain or unique atmospheric conditions such as underwater or high altitude B Predisposing risk factors include being very young elderly and in poor state of health and certain medications C Thermoregulation is the body s ability to ensure a balance between heat production and release The hypothalamus is the main organ in this regulation with the skin also having a major role D The body produces heat through metabolism with the basal metabolic rate BMR being the heat energy produced at rest from normal metabolic reactions Metabolism and body heat can be increased through exertion and absorption of environmental heat can also occur E Thermolysis is the release of heat and energy from the body while thermogenesis is the production of heat and energy for the body F The body has four main means of cooling itself Radiation transfer of heat to the environment Conduction transfer of heat to a cooler object through direct contact Convection loss of heat to air moving across the skin Evaporation conversion of liquid to a gas sweating G Heat illness is the increase in core body temperature CBT because of inadequate thermolysis the body cannot get rid of a heat buildup H Heat cramps are acute involuntary muscle pains in the abdomen or lower extremities caused by profuse sweating and sodium loss Pulse rate can be rapid with pale and moist skin and normal temperature Treatment includes moving to a cool environment providing a salt solution unless nauseated or normal saline IV I Heat syncope may occur when an overheated patient suddenly moves Treatment includes placing the patient supine and replacing fluids J Heat exhaustion can result from dehydration and heat stress with symptoms including headache fatigue dizziness nausea vomiting and abdominal cramping Skin is pale and clammy with rapid pulse rate and respirations Treatment includes removing patient from heat and providing fluids through sports drinks or IV K Heatstroke is a core temperature above F C and altered mental status Signs include changes in behavior nervous system disturbances elevated temperature tachycardia hyperventilation and dry red or pale and sweaty skin Treatment includes moving from the heat performing cooling measures normal saline and monitoring the cardiac system L Fever can mimic heatstroke so obtain a thorough history and treat for heatstroke if in doubt M Dress appropriately stay hydrated and stay in shade or air conditioning to prevent heat illness Community-based education programs aimed at high-risk populations can be valuable N Frostbite is the local freezing of a body part and is classified as superficial or deep Frostnip is a mild form O Superficial frostbite is characterized by numbness tingling or burning The skin is white waxy and firm to palpation When thawed the skin turns cyanotic and there is a hot stinging sensation Treatment includes getting the patient out of the cold rewarming the injured part with body heat and covering it with a warm sterile dressing and transporting the patient P Deep frostbite is characterized by white yellow-white or mottled blue-white injured body part and is hard cold and without sensation Thawing can cause major tissue damage Gangrene can necessitate amputation Treatment includes leaving the part frozen if found frozen or rewarming it if it is partially thawed Q Trench foot results from prolonged exposure to cool wet conditions Prevention is the best treatment R Hypothermia is a decrease in core body temperature and can be mild moderate or severe S Mild hypothermia is a core body temperature of greater than F C The patient shivers and may be confused have slurred speech or have impaired coordination Treatment includes passive rewarming such as removing wet clothing or drying the skin and providing warm fluids T Moderate hypothermia is a core body temperature from F to F C to C Treatment includes passive rewarming active external rewarming of truncal areas administering warmed IV fluids and potentially using special rewarming devices U Severe hypothermia is a core body temperature of less than F F Treatment includes active internal rewarming such as warm IV fluids and in-hospital measures V Resuscitate hypothermic patients who are not breathing or without a pulse Patients in cardiac arrest require CPR and possibly a single shock depending on heart rhythm follow local guidelines Attempt to insert an advanced airway ventilate with warm humidified oxygen and give IV fluids W Hypothermic patients with lethal traumatic injuries or patients so frozen as to block the airway or chest compression efforts are likely dead If a patient appears dead after prolonged exposure hypothermia may protect brain and other organs Resuscitation can be attempted in cases of cardiac arrest and hypothermia X Drowning is the process of experiencing respiratory impairment from submersion or immersion in liquid progressing from breath holding to laryngospasm to respiratory and cardiac arrest Y Rescuing a patient who has drowned should be undertaken by specially trained rescuers Treatment includes the ABCs and taking cervical spine precautions Positive end-expiratory pressure may keep the alveoli open and drive fluid out Insert a nasogastric tube to decompress the stomach if the patient is intubated Submersion patients may develop bronchospasm and may require beta- adrenergic drug administration Z For diving injuries obtain details such as type of diving type of tank number of dives in the past hours and predive and postdive activities AA Barotrauma can happen during dive descent from the pressure imbalance between the inside of the body and the outside atmosphere and may result in ear pain Treatment includes loose dressing for ear bleeding and IV antiemetics or sedatives BB Nitrogen narcosis is a state of altered mental status from breathing compressed air at depth Signs and symptoms include euphoria inappropriate foolish behavior and tingling of the lips gums and legs CC Pulmonary overpressurization syndrome POPS burst lung can occur if a diver ascends too quickly Signs and symptoms include mediastinal and subcutaneous emphysema sense of fullness in the throat pain on swallowing dyspnea and substernal chest pain DD Pulmonary overpressurization syndrome may lead to arterial gas embolism Air bubbles may travel to the coronary arteries causing cardiac arrest Symptoms include weakness or paralysis of the extremities seizures unresponsiveness and other neurologic symptoms EE Barotrauma treatment depends on whether there is an air embolism A pneumothorax may require needle decompression With an air embolism the patient must receive treatment in a hyperbaric chamber FF In decompression sickness nitrogen bubbles in blood and tissues during dive ascent symptoms include itchy skin subcutaneous emphysema swelling rashes joint and muscle pain sensory and motor disturbances incoordination paralysis chest pain and dyspnea Treatment is oxygen IV normal saline and transport to a hyperbaric facility GG Shallow water blackout occurs when a person hyperventilates just before diving and passes out before resurfacing Treatment is the same as for any submersion HH The Divers Alert Network is a resource for diving-related injuries Callers are immediately connected to a physician who can provide specific management advice II Altitude illness occurs when unacclimatized people ascend to altitude with types including acute mountain sickness AMS high-altitude cerebral edema HACE and high-altitude pulmonary edema HAPE JJ Symptoms of acute mountain sickness include headache and fatigue weakness gastrointestinal symptoms dizziness light-headedness and difficulty sleeping KK Symptoms of high-altitude cerebral edema include mental status changes and or ataxia in a person with acute mountain sickness or the presence of both in a person without acute mountain sickness LL Symptoms of high-altitude pulmonary edema include at least two of the following dyspnea at rest cough weakness or chest tightness or congestion and at least two of the following central cyanosis audible rales wheezing tachypnea or tachycardia MM Treatment of altitude illness includes descending or use of a portable hyperbaric chamber providing oxygen and administering IV medications NN Cardiopulmonary resuscitation should be started promptly for lightning strike victims OO In lightning strike cases victims who appear to be dead should be treated first reverse triage PP Anaphylactic reaction is the most frequent mortality cause from insect bites and envenomations QQ To decrease toxin exposure promptly remove hymenoptera stingers or venom sacs RR Fire ant stings may result in infection SS Most snake bites in the United States are from pit vipers crotalids TT If there are visible fang marks with no bleeding with a crotalid bite it is likely a dry bite with no venom UU For scene safety ensure the snake is dead gone or trapped in cases of envenomation VV Because all significant snake envenomations require antivenin treatment transport promptly The time between the bite and treatment is crucial WW The most concerning spider bites are from the female black widow the brown recluse and the hobo spider XX A small subset of patients with brown recluse spider bites may develop loxoscelism YY Scorpion stings produce a neurotoxic reaction causing autonomic excitation ZZ Scorpion sting treatment is largely supportive and includes airway protection AAA Tick bites can transmit serious illnesses and rarely cause life-threatening paralysis Treatment includes careful removal of the tick and washing the area around the bite Transport all patients with any neurologic symptoms Post-Lecture This section contains various student-centered end-of-chapter activities designed as enhancements to the instructor s presentation As time permits these activities may be presented in class They are also designed to be used as homework activities Assessment in Action This activity is designed to assist the student in gaining a further understanding of issues surrounding the provision of prehospital care The activity incorporates both critical thinking and application of paramedic knowledge Instructor Directions Direct students to read the Assessment in Action scenario located in the Prep Kit at the end of Chapter Direct students to read and individually answer the quiz questions at the end of the scenario Allow approximately minutes for this part of the activity Facilitate a class review and dialogue of the answers allowing students to correct responses as may be needed Use the quiz question answers noted below to assist in building this review Allow approximately minutes for this part of the activity You may wish to ask students to complete the activity on their own and turn in their answers on a separate piece of paper Answers to Assessment in Action Questions Answer A thermolysis Rationale The body reacts to hot environmental conditions by releasing stored heat and energy by a process called thermolysis Answer C hypothalamus Rationale Under normal circumstances temperature is regulated by the hypothalamus by a negative feedback system Answer D cutaneous vasoconstriction Rationale The body compensates for an increased core body temperature by cutaneous vasodilation Because of cutaneous vasodilation the effective volume of the vascular system is increased when the diameter of a tube such as an artery is increased its volume increases the heart must increase its output to compensate for this effect The pulse rate and stroke volume increase and the work of the heart is markedly increased Answer A perspiration sweat Rationale The only way the body can dissipate heat when the ambient temperature approaches body temperature is by the evaporation of sweat up to a point Answer B heatstroke Rationale Heatstroke is generally characterized by two main findings core temperature more than F C and altered mental status Answer C F C Rationale Cooling efforts should continue until the rectal temperature has fallen below about F C Answer D occur as rapidly as possible Rationale Cooling should occur as rapidly as possible by the most expeditious means available Answer B seizures Rationale Patients with heatstroke are at risk for seizures due to the severe rise in core body temperature Answer C covering the patient with wet sheets Rationale Covering the patient with wet sheets may impede heat loss by evaporation Additional Questions Rationale Obtaining a thorough history and performing a physical exam are vital to uncovering the cause of a patient s elevated temperature For example the presence of discolored and foul-smelling urine in an indwelling urinary catheter a recent complaint of cough and dyspnea a skin infection such as cellulitis or complaints consistent with meningitis may indicate an infectious agent Having a sound knowledge of disease processes and pathophysiology can assist the paramedic s critical thinking and clinical problem solving Rationale Elderly people often cannot generate heat effectively because of reduced muscle mass and a diminished shivering response Atrophy of subcutaneous fat also reduces elderly patients insulation against heat loss Medications commonly prescribed to older people may interfere with vasoconstriction as well Hypothyroidism and malnutrition may further contribute to an older person s vulnerability decreased thermogenesis Some elderly may also have limited access to heat during cold winter months due to budget issues Assignments A Review all materials from this lesson and be prepared for a lesson quiz to be administered date to be determined by instructor B Read Chapter Responding to the Field Code for the next class session Unit Assessment Keyed for Instructors Describe the body s thermoregulation system and the physiologic responses to heat and cold exposures Answer The hypothalamus in the brain is responsible for the thermoregulation or balancing of heat production and dissipation in the body It functions under the control of negative feedback such that a rise in core body temperature elicits responses that will increase heat loss and shut off normal heat production pathways This process is thermogenesis and it is the main method of dealing with cold stressors The hypothalamus and sympathetic nervous system increase muscle tone and initiate shivering The hypothalamus can also stimulate peripheral vasoconstriction and eccrine sweat glands receive stimuli to decrease sweating If the core body temperature falls thermolysis occurs During thermolysis heat production and conservation is prompted while normal heat-liberating pathways are turned off There are four major ways for the body to cool radiation or the transfer of heat via electromagnetic waves conduction or the transfer of heat from a hotter object to a cooler one by direct physical contact convection or the transfer of energy when moving air or liquid disturbs molecules next to an object and evaporation or the conversion of a liquid to a gas Metabolic functions within the body work best when the normal constant core body temperature is maintained pp - Differentiate between the three primary heat-related illnesses including signs or symptoms and management Answer Heat cramps are acute involuntary painful muscle spasms that typically occur in the lower extremities abdomen or both They occur as a result of profuse sweating that results in sodium loss Patients with existing salt depletion dehydration and muscle fatigue are at higher risk Heat cramps are commonly seen in patients in good physical condition These patients will have a normal mental status with a potentially mildly elevated body temperature and cool moist skin Management includes eliminating exposure and restoring lost salt and water Heat exhaustion is a clinical syndrome that is considered to be a milder form of heat illness but may lead to heat stroke These patients will present with volume depletion and heat stress There are two forms water depleted and sodium depleted Geriatric patients commonly have water-depleted heat exhaustion due to immobility medications that contribute to dehydration and decreased sensitivity to thirst Other groups that do not adequately rehydrate in a hot environment are at risk for water-depleted heat exhaustion It may take hours or days to develop sodium-related heat exhaustion These patients may present with normal or mild confusion mildly elevated body temperatures and pale cool and moist skin Muscle cramping may or may not be present These patients should be moved to a cool environment and placed in a supine position with legs elevated and excess clothing should be removed Tepid water may be sponged sprayed or dripped onto the patient Oral hydration may be adequate but IV fluid replacement may be necessary Heat stroke while the least common

Related Downloads
Explore
Post your homework questions and get free online help from our incredible volunteers
  1036 People Browsing
Your Opinion
What's your favorite math subject?
Votes: 314